LSAT and Law School Admissions Forum

Get expert LSAT preparation and law school admissions advice from PowerScore Test Preparation.

 Administrator
PowerScore Staff
  • PowerScore Staff
  • Posts: 8919
  • Joined: Feb 02, 2011
|
#38111
Complete Question Explanation
(The complete setup for this game can be found here: lsat/viewtopic.php?t=14969)

The correct answer choice is (E)

This question stipulates that F is evaluated by R. From the second rule, we know that R would also evaluate L, because F and L are evaluated by the same officer. Additionally, from our original diagram we can conclude that the only officer who can evaluate K is T:
PT71_D13 LG Explanations_Game #2_#11_diagram 1.png
With three applicants remaining (H, I, and M), S must evaluate at least two of them in compliance with the last rule. However, the third rule prohibits the same officer from evaluating both H and I, and it also prohibits the same officer from evaluating both M and I. Therefore, S cannot evaluate I, and must evaluate H and M instead:
PT71_D13 LG Explanations_Game #2_#11_diagram 2.png
The question stem asks for how many of the applicants (other than F) is the identity of the officer who evaluates it fully determined. Since exactly five of our variables (not including F) are placed squarely in their respective groups—L, H, M, K, and G—the correct answer choice is (E).
 cardinal2017
  • Posts: 19
  • Joined: Oct 23, 2016
|
#34448
Hi there,

I chose (D) at first for this question -

but it was just because there was no option of seven among the answer choices.

R: F L
S: _ _ (_)
T: K
U: G (_)

The basis (framework) for my decision for (D) is as the above diagram.

However, I knew according to the above diagram alone, you can determine 'I'(for Inman) should go to U and both H&M should, therefore, go to S.

So my actual answer would be seven but since there's no seven in the AC options, I just went for FOUR with dissatisfaction.

Anyone could help me?

Thanks in advance!
 Luke Haqq
PowerScore Staff
  • PowerScore Staff
  • Posts: 722
  • Joined: Apr 26, 2012
|
#34468
Hi cardinal2017!

I can certainly help you with this one--I'll go through how I tackled the problem.

As a first point, the game refers to the "applications of seven job candidates." However, you wrote,
So my actual answer would be seven but since there's no seven in the AC options, I just went for FOUR with dissatisfaction.
Since the question asks for how many others can be precisely determined, the answer can't be 7, because there are only 6 others.

In addressing the substance of the question, you definitely had the basis of the diagram right:

R: F L
S:
T: K
U: G

At this point, we're left with three variables--M, I, and H. We know from the rules that S must evaluate more applications than T, so there must be either 2 or 3 variables that go in S. From there, we know it must be two variables that go in S--these must be H and M.

In sum, if we're told that F is evaluated by R, we can know the precise positions of L, M, H, K, and G--five others, as indicated by answer (E).

Hope that helps!
 jwheeler
  • Posts: 39
  • Joined: Aug 19, 2018
|
#59885
I got the connection of how all the rules play into each other in this one (S has to evaluate more than T, H&M must be together since neither can be with I, etc.). My issue was with the counting of variables. I didn't include G in my count (so I went with 4 instead of 5) because we're told that G is evaluated by U. I thought that was just considered a given; F being evaluated by R has no real bearing of the placement of G. It will ALWAYS be evaluated by U. That was tricky to me and I considered the possibility of them counting G (and therefore 5 being the correct answer), but I thought my reasoning was sound. Any thoughts or ways to avoid this trap in the future?
 Robert Carroll
PowerScore Staff
  • PowerScore Staff
  • Posts: 1787
  • Joined: Dec 06, 2013
|
#61557
jwheeler,

Nothing in the question asks about how many other officers are determined by that fact alone. It's simply asking how many officer identifies are determined total. That determination may be due to previous conditions - nothing in the question says otherwise. So you just count how many are determined, and don't consider the source of the determination.

It's difficult to think of a question that would be asked in a way that would exclude G. I don't think questions on the test would ever do that. So you can pretty much rest easy evaluating such questions as asking for the total count (other than F, of course).

Robert Carroll
 ncolicci11
  • Posts: 43
  • Joined: Feb 09, 2020
|
#75781
Powerscore,

I had exactly the diagram for this question but selected four because G was already placed in U in the rules. Even though the rules gave us G and U in a block, this still counts for what we can determine?
 Jeremy Press
PowerScore Staff
  • PowerScore Staff
  • Posts: 1000
  • Joined: Jun 12, 2017
|
#75859
Hi ncolicci,

Yes! Because the question stem asks for the identity of the officers evaluating applications "other than" F's application, G is included as a variable we have to think about. Since the identity of the officer evaluating G's application is "fully determined," it counts. Doesn't matter whether G is fully determined because of the local condition or the global conditions, it's still fully determined!

Hope this helps!

Jeremy

Get the most out of your LSAT Prep Plus subscription.

Analyze and track your performance with our Testing and Analytics Package.